You are on page 1of 34

ARML Competition 2022

George Reuter, Head Writer


Chris Jeuell, Lead Editor
Evan Chen
Thinula De Silva
Paul Dreyer
Edward Early
Zuming Feng
Zachary Franco
Silas Johnson
Winston Luo
Jason Mutford
Andy Niedermaier
Graham Rosby
Tristan Shin
April Verser

June 3–4, 2022

Sponsored By:

ARML encourages the reproduction of our contest problems for non-commercial, educational purposes.
Commercial usage of ARML problems without permission and posting entire contests or contest books are prohibited.
1 Team Problems

Problem 1. Compute the number of ordered pairs of positive integers (m, n) such that m ≤ n2 and m + n ≤ 53.

Problem 2. The base of a pyramid is a regular decagon of side length 2 and area B. The height of the pyramid
is 1. The total surface area of the pyramid is L + B. Compute cos−1 B
L in degrees.

Problem 3. Compute the positive real value of x such that



3

20x + 22 − 3 20x − 22 = 2.

Problem 4. Let S be the infinite sum of the reciprocals of the positive cubes, and let T be the infinite sum of the
reciprocals of the positive cubes that are neither multiples of 2 nor 3. That is,
1 1 1 1 1 1 1 1
S= + 3 + 3 + 3 + ··· and T = + 3 + 3 + 3 + ··· .
13 2 3 4 13 5 7 11
S
Compute .
T

Problem 5. Tom has a positive integer n. He computes the sum of the digits of n in base 10 and writes it down
(e.g., if n = 2310 , he would write down 5). He then converts n to base 8 and computes the sum of the digits of
n in base 8, expressing the sum in base 8 (e.g., if n = 2310 = 278 , he would write down 11 because 2 + 7 = 118 ).
To his surprise, he has written down 10 both times. Compute the least possible value of n; express your answer
in base 10.

Problem 6. Four positive integers have the property that no two of them are relatively prime, but any three of
them are mutually relatively prime. Compute the least possible value of the sum of the four integers.

Problem 7. A 6×6 board has rows and columns numbered from 1 through 6. Each cell of the board is labeled with
a pair of numbers denoting the cell’s row and column. Two cells labeled (r1 , c1 ) and (r2 , c2 ) are called strictly
comparable if r1 < r2 and c1 < c2 . Three different cells are chosen on the board. Compute the probability that
no two of the three chosen cells are strictly comparable in either order.

Problem 8. Let S = {2, 3, 4, 5, 6, 7, 8}. Compute the number of ways to select an ordered pair of nonempty sets
(A, B) such that A ⊆ S, B ⊆ S, and each element of A is relatively prime to each element of B.

Problem 9. Two circles intersect at points A and B. A line is tangent to the two circles at points X and Y . The
←→
circle with diameter XY meets AB at points P and Q. Points Q, A, P , and B are collinear in that order, and
P B = 20 and P A = 19. Given that the lengths P Q, P X, and P Y are integers, compute the average of these
three lengths.

Problem 10. Let M be the product (101)(102)(103) · · · (505). Danielle writes M in decimal form on a chalkboard,
then erases all occurrences of the digit zero to obtain a new number N . Compute the remainder when N is
divided by 1000.

ARML encourages the reproduction of our contest problems for non-commercial, educational purposes. 1
Commercial usage of ARML problems without permission and posting entire contests or contest books are prohibited.
2 Answers to Team Problems

Answer 1. 1172

Answer 2. 18 (or 18◦)



7
Answer 3.
2
108 17
Answer 4. (or 1 91 )
91
Answer 5. 1072

Answer 6. 718
229
Answer 7.
714
Answer 8. 438

Answer 9. 608

Answer 10. 712

2 ARML encourages the reproduction of our contest problems for non-commercial, educational purposes.
Commercial usage of ARML problems without permission and posting entire contests or contest books are prohibited.
3 Solutions to Team Problems

Problem 1. Compute the number of ordered pairs of positive integers (m, n) such that m ≤ n2 and m + n ≤ 53.

Solution 1. Note that n ≤ 52. If n2 + n ≤ 53 (so 1 ≤ n ≤ 6), then 1 ≤ m ≤ n2 are solutions. Otherwise, if
7 ≤ n ≤ 52, then 1 ≤ m ≤ 53 − n are solutions. Therefore the number of solutions is given by:
6 52 46
X X 6 · 7 · 13 X 46 · 47
n2 + (53 − n) = + n = 91 + = 1172.
n=1 n=7
6 n=1
2

Hence there are 1172 ordered pairs of positive integers (m, n) such that m ≤ n2 and m + n ≤ 53.

Problem 2. The base of a pyramid is a regular decagon of side length 2 and area B. The height of the pyramid
is 1. The total surface area of the pyramid is L + B. Compute cos−1 B
L in degrees.

2 2
Solution 2. Let r and R be the inradius and circumradius of the √ base, respectively. Then r + 1 = R . The area
of the base is 10r, and 2
 the height of each
360◦
triangular face is 1 + r = R. Therefore the lateral surface area is
10R. Thus cos−1 B L = cos−1 r
R = 20 = 18 .

Problem 3. Compute the positive real value of x such that



3

20x + 22 − 3 20x − 22 = 2.

√ √
Solution 3. Let M = 3 20x + 22 and N = 3 20x − 22. Then the given equation is equivalent to M − N = 2,
which implies (M − N )3 = M 3 − 3M 2 N + 3M N 2 − N 3 = 8. This is equivalent to (20x + 22) − 3M 2 N +
3M N 2 − (20x − 22) = 8 → 44 − 3M N (M − N ) = 8, so M N (M − N ) = 12. The problem statement implies
that M − N = 2, so M N = 6. Cubing both sides again√
yields M 3 N 3 = 216 → (20x + 22)(20x − 22) = 216,
2 7
which implies 400x = 700, and so the answer is x = 2 .

Problem 4. Let S be the infinite sum of the reciprocals of the positive cubes, and let T be the infinite sum of the
reciprocals of the positive cubes that are neither multiples of 2 nor 3. That is,
1 1 1 1 1 1 1 1
S= 3
+ 3 + 3 + 3 + ··· and T = 3
+ 3 + 3 + 3 + ··· .
1 2 3 4 1 5 7 11
S
Compute .
T

Solution 4. For integers n ≥ 2, let


1 1 1 1
Sn = + + + + ··· .
n3 (2n)3 (3n)3 (4n)3
That is, Sn is the sum of the reciprocals of the cubes of the positive multiples of n. Note that T excludes all
summands of S2 and S3 ; however S − S2 − S3 subtracts the summands of S6 = S2·3 twice, so it follows that
T = S − S2 − S3 + S6 . Also note that
Å ã
1 1 1 1 1 S
Sn = 3 3 + 3 + 3 + 3 + · · · = 3 .
n 1 2 3 4 n
Thus Å ã
S S S 1 1 1 91
T = S − S2 − S3 + S6 = S − 3 − 3 + 3 = S 1 − − + =S· ,
2 3 6 8 27 216 108

ARML encourages the reproduction of our contest problems for non-commercial, educational purposes. 3
Commercial usage of ARML problems without permission and posting entire contests or contest books are prohibited.
S 108
and hence = .
T 91

Problem 5. Tom has a positive integer n. He computes the sum of the digits of n in base 10 and writes it down
(e.g., if n = 2310 , he would write down 5). He then converts n to base 8 and computes the sum of the digits of
n in base 8, expressing the sum in base 8 (e.g., if n = 2310 = 278 , he would write down 11 because 2 + 7 = 118 ).
To his surprise, he has written down 10 both times. Compute the least possible value of n; express your answer
in base 10.

Solution 5. Denote the sum of the digits of n in base b by sb (n). The problem statement implies s10 (n) = 10 and
s8 (n) = 108 = 810 . Because s10 (n) ≡ n (mod 9) and s8 (n) ≡ n (mod 7), it follows that n ≡ 1 (mod 9) and
n ≡ 1 (mod 7). By the Chinese Remainder Theorem, it follows that n ≡ 1 (mod 63).

Observe that 6310 = 778 . The first few positive integers that are congruent to 1 (mod 63) are 18 and 1008 , nei-
ther of which works. The next few positive integers that are congruent to 1 (mod 63) are 1778 , 2768 , 3758 , . . . , 7718 ,
none of which works.

The next candidate is 10708 . But this is 56810 , so this does not work. The next few positive integers that are
congruent to 1 (mod 63) are 11678 , 12668 , 13658 , . . . , 17618 , none of which works.

The next candidate is 20608 . This is 107210 , which has sum of digits 10, so the answer is 1072.

Alternate Solution: In general, when an integer n is written in base b, the sum of its digits is congru-
ent to n modulo b − 1, and the alternating sum of the digits (i.e., starting from the units digit and alternately
adding and subtracting digits) is congruent to n modulo b + 1. Because the sum of the digits of Tom’s number
is 10 when written in base 10, it follows that n ≡ 1 (mod 9). This, in turn, implies that the alternating sum
of the digits of n when written in base 8 is congruent to 1 (mod 7). Meanwhile, the sum of the digits of n in
base 8 is 108 = 810 . Note that the alternating sum of the digits cannot be greater in absolute value than the
sum, and must be of the same parity as the sum, and thus must be equal to −8. The two least positive integers
that have a digit sum of 8 and an alternating digit sum of −8 when written in base 8 are 10708 = 56810 and
20608 = 107210 . Only the latter has a digit sum of 10 when written in base 10, so the least possible value of n
is 1072.

Problem 6. Four positive integers have the property that no two of them are relatively prime, but any three of
them are mutually relatively prime. Compute the least possible value of the sum of the four integers.

Solution 6. Let the integers be a1 , a2 , a3 , a4 , and let S = a1 + a2 + a3 + a4 . Consider the greatest common
divisors for the six pairs {ai , aj } of the four integers. Suppose the one of these is composite, and suppose that
gcd(ai , aj ) = pq for some i and j and some prime p and some integer q. Then replacing ai and aj with api and
aj
p creates a different set of four positive integers that satisfies the conditions of the problem and has a lesser sum.

Thus suppose that the greatest common divisor of each pair of the four positive integers is prime. That
is, let gcd(ai , aj ) = pij = pji . Because any three ak are relatively prime, it follows that these six primes must
be distinct. The six least primes are 2, 3, 5, 7, 11, 13, and the problem requires a minimum sum S, where
S = p12 p13 p14 + p21 p23 p24 + p31 p32 p34 + p41 p42 p43 .

Note that 2S = (p12 + p34 )(p13 + p24 )(p14 + p23 ) + (p12 − p34 )(p13 − p24 )(p14 − p23 ).

The factors in (p12 + p34 )(p13 + p24 )(p14 + p23 ) have a fixed sum, so their product is minimized when they are as
far apart as possible. Thus it follows that the minimal product is (2 + 3)(5 + 7)(11 + 13) = 1440. The product
(p12 − p34 )(p13 − p24 )(p14 − p23 ) ≥ −1 · 2 · 2 = −4, so 2S ≥ 1440 − 4 = 1436, and the answer is S = 718.

4 ARML encourages the reproduction of our contest problems for non-commercial, educational purposes.
Commercial usage of ARML problems without permission and posting entire contests or contest books are prohibited.
Problem 7. A 6×6 board has rows and columns numbered from 1 through 6. Each cell of the board is labeled with
a pair of numbers denoting the cell’s row and column. Two cells labeled (r1 , c1 ) and (r2 , c2 ) are called strictly
comparable if r1 < r2 and c1 < c2 . Three different cells are chosen on the board. Compute the probability that
no two of the three chosen cells are strictly comparable in either order.

Solution 7. Count the number of triples with at least one strictly comparable pair.

Case 1: Suppose the set of numbers r1 , r2 , r3 has two elements {a < b} and the set of numbers c1 , c2 ,
c3 has two elements {d < e}. Then there are two possible triples: (a, d), (a, e), (b, d) and (b, d), (b, e), (a, e).
This results in 2 · 62 62 favorable outcomes.
 

Case 2: Suppose the set of numbers r1 , r2 , r3 has three elements {a < b < c} and the set of numbers
c1 , c2 , c3 has three elements {d < e < f }. Then there are 3! − 1 = 5 five possible triples because among the
3! = 6 ways to permute the ri -values, there is only 1 such permutation for which the pairs of cells are not
strictly comparable. These are: (a, d), (b, e), (c, f ); (a, d), (b, f ), (c, e); (a, e), (b, d), (c, f ); (a, e), (b, f ), (c, d);
and (a, f ), (b, d), (c, e). This results in 5 · 63 63 favorable outcomes.
 

Case 3: Suppose the set of numbers r1 , r2 , r3 has two elements {a < b} and the set of numbers   c1 , c2 ,
c3 has three elements {d < e < f }. Then there are four possible triples, and this results in 4 · 63 62 favorable
outcomes.

Case 4: Suppose the set of numbers r1 , r2 , r3 has three elements {a < b < c} and the set  of
 numbers
c1 , c2 , c3 has two elements {d < e}. Then there are four possible triples, and this results in 4 · 63 62 favorable
outcomes.
6
 6 6
 6 6
 6
The number of favorable outcomes is 2 · 2 2 +8· 3 2 +5· 3 3 = 4850. Because this argument uses
complementary counting, the answer is
4850 229
1− 36 = 714 .

3

Alternate Solution: Think of the cells as points Pi = (xi , yi )i=1,2,3 and assume without loss of generality
that they are lexicographically ordered (i.e., if i < j, then xi ≤ xj , and if xi = xj , then yi < yj ). The condition
is that none of P1 P2 , P1 P3 , or P1 P3 has positive slope. Fix P2 = (m, n) and consider the possible locations
for P1 and P3 . Note that 1 ≤ x1 ≤ m and n ≤ y1 ≤ 6, and similarly note that m ≤ x3 ≤ 6 and 1 ≤ y3 ≤ n.
Making sure that the cases where P1 = P2 and P2 = P3 are not counted, the number of possible pairs (P1 , P3 )
is (m(7 − n) − 1)(n(7 − m) − 1). Sum over all possible P2 to obtain

6 X
6 6
!2 6 6
X X X X
(m(7 − n) − 1)(n(7 − m) − 1) = m(7 − m) −2 m (7 − m) + 36,
m=1 n=1 m=1 m=1 m=1

which equals (2(1 · 6 + 2 · 5 + 3 · 4))2 − 2 · 212 + 36 = 562 − 2 · 212 + 36 = 2290.


2290 229
Thus the desired probability is 36 = 714 .

3

Problem 8. Let S = {2, 3, 4, 5, 6, 7, 8}. Compute the number of ways to select an ordered pair of nonempty sets
(A, B) such that A ⊆ S, B ⊆ S, and each element of A is relatively prime to each element of B.

Solution 8. Consider the modified problem of counting the number of ordered pairs of sets (A, B) that satisfy the
conditions of the problem but where either set can be empty. There are two cases to consider.

ARML encourages the reproduction of our contest problems for non-commercial, educational purposes. 5
Commercial usage of ARML problems without permission and posting entire contests or contest books are prohibited.
Case 1: 6 ∈ A ∪ B. There are 2 possibilities for the set that contains 6. That set may also contain any of
2, 3, 4, 8, so there are 2 · 24 possible assignments for the numbers 2, 3, 4, 6, 8. Now either 5 ∈ A, 5 ∈ B, or
5∈/ A ∪ B (i.e., there are 3 possible assignments for the element 5). By an analogous argument, the same is
true for 7, hence there are 32 possible assignments for the numbers 5 and 7. This gives a total of 2 · 24 · 32 = 288
possible ordered pairs (A, B).

Case 2: 6 ∈ / A ∪ B. Each of 3, 5, 7 can belong to either A or B with no further restrictions, and any elements
of {2, 4, 8}, if present in either set, must belong to the same set. In this case there are 33 · (23 · 2 − 1) = 405
possible ordered pairs (A, B). The factor of 33 comes from the fact that each of 3, 5, 7 can either belong to A,
or to B, or to neither A nor B, hence 3 · 3 · 3. The factor of 23 · 2 − 1 comes from selecting a subset of {2, 4, 8}
(23 possible subsets), assigning it to either A or B (2 possibilities), and then subtracting 1 because this counts
the empty set assignment to both sets twice.

Thus the answer to the modified problem is 288 + 405 = 693. Returning to the original problem, the or-
dered pairs (A, B) where at least one of A, B is empty must be excluded. The number of such ordered pairs
is 2 · 27 − 1 = 255, where 1 is subtracted so as not to count (∅, ∅) twice. Thus the answer is 693 − 255 = 438.

Problem 9. Two circles intersect at points A and B. A line is tangent to the two circles at points X and Y . The
←→
circle with diameter XY meets AB at points P and Q. Points Q, A, P , and B are collinear in that order, and
P B = 20 and P A = 19. Given that the lengths P Q, P X, and P Y are integers, compute the average of these
three lengths.

Solution 9. Let line AB meet line XY at O. Then by Power of a Point, it follows that OX 2 = OY 2 = OA · OB.
Because m∠XP Y = 90◦ , it follows that OP = OX = OY . Let r = OX = OY = OP ; this equation then
implies
r2 = OA · OB = (r − 19)(r + 20) = r2 + r − 380,
so r = 380, and hence XY = 2r = 760.

Y
O
X
A

It remains to determine P X and P Y . By the Pythagorean Theorem,


P X 2 + P Y 2 = 7602 = 26 · 52 · 192 . (∗)
The following argument shows that the only possibility is {P X, P Y } = {8 · 19 · 3, 8 · 19 · 4}. First, taking (∗)
modulo 19 gives 19 | P X and 19 | P Y (because 19 is a prime congruent to 3 modulo 4, there are no nontrivial

6 ARML encourages the reproduction of our contest problems for non-commercial, educational purposes.
Commercial usage of ARML problems without permission and posting entire contests or contest books are prohibited.
solutions to a2 + b2 ≡ 0 (mod 19)). Furthermore, taking (∗) modulo 4 implies that both P X and P Y are
even (because odd squares are congruent to 1 modulo 4); dividing out this common factor of 2 and repeating
eventually shows that 8 | P X and 8 | P Y . Letting x = 8 · 19 · P X and y = 8 · 19 · P Y and substituting into (∗)
gives x2 + y 2 = 25, and the only possible positive integral solution is {x, y} = {3, 4}.

1
In conclusion, the answer is 3 · 8 · 19 · (5 + 3 + 4) = 608.

Note: The configuration described in the problem does in fact exist. To construct it (and thus verify that
{P X, P Y } = {8 · 19 · 3, 8 · 19 · 4} is a solution), begin by drawing a circle of radius 380 centered at O. Mark
points X, Y , and P on the circumference so that XY is a diameter and 4P XY is a 3−4−5 triangle with
←→
right angle at P ; this gives P X and P Y the desired lengths. Then draw line OP , and let Q be the other point
where this line meets the circle. Mark off points A and B on the line at the required locations, i.e., such that
AP = 19, BP = 20, A is between P and Q, and B is not between P and Q. Then let ω1 and ω2 be the
circumcircles of 4XAB and 4Y AB, respectively. It remains to verify that ω1 and ω2 are tangent to XY at
X and at Y . Because OX 2 = OA · OB = OY 2 , it follows by Power of a Point that XY is tangent to both
circles, as desired.

Problem 10. Let M be the product (101)(102)(103) · · · (505). Danielle writes M in decimal form on a chalkboard,
then erases all occurrences of the digit zero to obtain a new number N . Compute the remainder when N is
divided by 1000.

Solution 10. First, consider the product


A = (102)(103) · · · (505).
This solution will find the last three nonzero digits of this number.

505!
Denote the number of factors of k in the prime factorization of n by νk (n). Notice that A = 101! . Let B be
101
the product of all integers between 1 and 505 inclusive that are not divisible by 5. Then 505! = 5 · 101! · B.
505!
Thus A = 101! = 5101 · B, and so it follows that ν5 (A) = 101. Furthermore, note that

A Y
B= = k
5101
1≤k≤504
gcd(k,5)=1

because for every such k, there is exactly one e with 5e k ∈ [102, 505].

Now, using the fact that (5n + 1)(5n + 2)(5n + 3)(5n + 4) ≡ 24 + 50n (mod 125), it follows that
100
Y
B≡ (24 + 50n) (mod 125)
n=0
101
≡ 24 + 50 · 24100 (1 + · · · + 100) (mod 125)
≡ 24 (mod 125)
A
from the fact that x100 ≡ 1 (mod 125) whenever gcd(x, 5) = 1. Next let C = 10101 , and note that
Å ã101
A 1 1
C = 101 = · B ≡ B ≡ 12 (mod 125).
10 2 2

Therefore C ≡ 512 (mod 1000) because C ≡ 0 (mod 8).

Finally, 101 · C ≡ 712 (mod 1000). Thus the desired remainder is 712.

ARML encourages the reproduction of our contest problems for non-commercial, educational purposes. 7
Commercial usage of ARML problems without permission and posting entire contests or contest books are prohibited.
4 Power Question 2022: June Madness
Instructions: The power question is worth 50 points; each part’s point value is given in brackets next to the part.
To receive full credit, the presentation must be legible, orderly, clear, and concise. If a problem says “list” or “com-
pute,” you need not justify your answer. If a problem says “determine,” “find,” or “show,” then you must show
your work or explain your reasoning to receive full credit, although such explanations do not have to be lengthy. If a
problem says “justify” or “prove,” then you must prove your answer rigorously. Even if not proved, earlier numbered
items may be used in solutions to later numbered items, but not vice versa. Pages submitted for credit should be
NUMBERED IN CONSECUTIVE ORDER AT THE TOP OF EACH PAGE in what your team considers to be
proper sequential order. PLEASE WRITE ON ONLY ONE SIDE OF THE ANSWER PAPERS. Put the TEAM
NUMBER (not the team name) on the cover sheet used as the first page of the papers submitted. Do not identify
the team in any other way.

ARML is organizing a Sportsball competition, which consists of a regular season and a playoff tournament. Each
phase consists of a series of games; each game involves two teams, and results in one team winning and the other
team losing (there are no ties). Assume that in all games, the teams are evenly matched, so each team playing has
an equal chance of winning each game.
The Regular Season

In the regular season, the organizers choose two positive integers W and L. Each team will qualify for the playoffs
if and only if it wins W games before it loses L games. That is, the team is eliminated from playoff contention
upon losing L games, and qualifies upon winning W games, whichever comes first. Note that the number of teams
qualifying for the playoffs is not necessarily fixed.
1. Before the Sportsball starts, the referee will make the opening coin tosses.
a. On Opening Day, the referee flips a fair coin five times. Compute the probability that at least four of
these flips will be heads. [2 pts]
b. For all other games, the referee flips a fair coin until she flips five heads. For one of these games, compute
the probability that she will flip two or fewer tails. [2 pts]
2. Now suppose Team X is one of the teams competing in the Sportsball competition.
a. Compute the probability that Team X qualifies for the playoffs with W = 4 and L = 2. [1 pt]
b. Compute the probability that Team X qualifies for the playoffs with W = 5 and L = 2. [1 pt]
c. Compute the probability that Team X qualifies for the playoffs with L = 2, in terms of W . [2 pts]
3. Suppose W = 10 and L = 2.
a. Compute the probability that Team X will not be eliminated after playing five games. [2 pts]
b. Compute the probability that Team X will not be eliminated after playing six games. [2 pts]
4. Suppose there are a total of N teams in the competition and W > 5.
a. Find in terms of N the expected number of teams that win their first five games. [2 pts]
b. Given that L = 2, compute the expected number of teams that have not been eliminated after their first
five games. [1 pt]
c. Given that L = 2, compute, in terms of N and W , the expected number of teams that qualify for the
playoffs. [2 pts]

8 ARML encourages the reproduction of our contest problems for non-commercial, educational purposes.
Commercial usage of ARML problems without permission and posting entire contests or contest books are prohibited.
For Problems 5 and 6, let L = 2.
5. Suppose a team has won a games and lost b games, with either a < W or b < L. Let V (W, a, b) be the
probability that this team qualifies for the playoffs. For example, V (W, a, 2) = 0, and V (W, W − 1, 1) = 21 .

a. Compute V (5, 2, 0). [2 pts]


b. Find a formula for V (W, a, 1) in terms of W and a. [2 pts]
c. Find a formula for V (W, a, 0) in terms of W and a. [2 pts]
6. Show that V (W, a + 1, b) − V (W, a, b) = V (W, a, b) − V (W, a, b + 1). [3 pts]

The Playoffs

The playoff tournament consists of a series of games, played one at a time, so that the results of each game can affect
which teams play in future games. The playoffs are double-elimination, meaning that a team is eliminated when it
loses its second playoff game, playing no further games afterwards. (Regular-season results do not carry over, and
each team starts the playoffs with 0 wins and 0 losses.) The last team remaining wins the tournament.
Suppose that T teams qualify for the playoffs. The organizers wish to arrange the playoff games in such a way that
for some W > 1, it is guaranteed that the eventual winner of the tournament has won exactly W playoff games,
regardless of who wins each game (receiving a “bye” or otherwise not playing a game never counts as a win). If
the games are arranged in this way, call the tournament W -perfect, and if a W -perfect tournament with T players
exists, call (T, W ) a perfect pair. For example, (2, 2) is a perfect pair, and the corresponding 2-perfect tournament
is a “best-of-3” series between the two teams, whereas (2, 3) is not a perfect pair.
7. Show that if the tournament is W -perfect, then the probability that any given team wins the tournament is
W +2
equal to W +1 . [3 pts]
2
8. Show that if (T, W ) is a perfect pair, then T and W + 2 must both be powers of 2. [3 pts]
In all of the following problems, assume that the playoff tournament is W -perfect, T = 2k , and W + 2 = 2r , where
k and r are integers such that k ≥ 1 and r ≥ 2.

9. Find a formula for k in terms of r. [3 pts]


The following information will be useful for Problems 10–12. Suppose that a team has thus far won a games and
lost b games.
• Define the team’s value V (W, a, b) by the formula you found in Problem 5(b) (if b = 1) or Problem 5(c) (if
b = 0).
• Define the team’s stake by S(W, a, b) = V (W, a + 1, b) − V (W, a, b) (i.e., the quantity in Problem 6).
It can be shown that no matter which teams play in which games, then it is possible for exactly one team to win its
first k games. Furthermore, if k ≤ t ≤ W , then it can be shown that it is possible for exactly one team to win its
first t games, and for no other team to win its first k games. It can also be shown that whenever two teams play
each other in the Playoffs, they must have equal stakes. You may use these facts without proof in the problems that
follow.
10. Show that for each integer t with k ≤ t < W , there exists an integer t0 such that S(W, t, 0) = S(W, t0 , 1).
[4 pts]
11. Prove that W must equal either 2 or 6. [5 pts]
12. Prove that (16, 6) is a perfect pair, by showing how to arrange the games in a 6-perfect tournament between
16 teams. [6 pts]

ARML encourages the reproduction of our contest problems for non-commercial, educational purposes. 9
Commercial usage of ARML problems without permission and posting entire contests or contest books are prohibited.
5 Solutions to Power Question
1 5 1 5 1 5
  
1. a. The probability of flipping five heads is 2 = 32 ,
and the probability of flipping four heads is 4 · 2 =
5 6 3
32 , so the total probability of flipping at least four heads is 32 = 16 .

b. The referee flips two or fewer tails if and only if she flips five heads in seven or fewer flips. The probability
of doing so is the same as the probability of flipping five or more heads in seven total flips, which is equal
to ÇÇ å Ç å Ç åå
7 7 7 1 29
+ + · 7 = .
0 1 2 2 128

2. a. In order to qualify, Team X must win four games before losing two. This is the same as the probability
3
of flipping a fair coin five times and obtaining four or more heads, which from Problem 1(a) is .
16
b. This is the same as the probability of flipping a fair coin six times and obtaining five or more heads.
6+1 7
Following the same logic as in Problem 1(a), this is 6
= .
2 64
c. In general, this is equal to the probability of flipping a fair coin W + 1 times and obtaining at least W
W +2
heads, which is W +1 .
2

3
3. a. The answer is , the same as the answer to Problem 2(a), because not being eliminated after five
16
games requires winning at least four of those five games. The fact that W = 10 is irrelevant.
7
b. The answer is , the same as the answer to Problem 2(b), because not being eliminated after six games
64
requires winning at least five of those six games. The fact that W = 10 is irrelevant.
5
4. a. The probability that any one team wins its first five games is 12 , so the expected number of teams that
N
do so is .
32
3
b. From Problem 3(a), the probability that any one team remains in contention after five games is 16 , so the
3N
expected number of teams that do so is .
16
W +2
c. From Problem 2(c), the probability that any one team qualifies for the playoffs is 2W +1
, so the expected
N (W + 2)
number of teams that do so is .
2W +1
5. a. This team needs 3 more wins to qualify for the playoffs, so V (5, 2, 0) is the probability of flipping 3 or
4+1 5
more heads on 4 attempts (i.e., winning 3 games before losing 2). This is 4
= .
2 16
1
b. This team must win its next W − a games without losing any, the probability of which is .
2W −a
c. This team must win W − a games with at most one loss; its probability of doing so is the same as the
W −a+2
probability of flipping W − a heads or more on W − a + 1 attempts, which is .
2W −a+1
6. This can be shown by brute force using the formulas in Problems 5(b) and 5(c). A more elegant approach,
however, is to recognize that a team with a wins and b losses has equal probabilities, after one more game, to

10 ARML encourages the reproduction of our contest problems for non-commercial, educational purposes.
Commercial usage of ARML problems without permission and posting entire contests or contest books are prohibited.
have either a + 1 wins and b losses, or a wins and b + 1 losses. Therefore
1 
V (W, a, b) = V (W, a + 1, b) + V (W, a, b + 1) .
2
Multiplying by 2 and rearranging gives the desired equality.
7. From the perspective of just one team, the probability of winning is the same as the probability, in the regular
W +2
season, of winning W games before losing 2. This is the same as the answer to Problem 2(c), which is W +1 ,
2
as desired.
W +2 1
8. By symmetry, each team must have equal probability of winning the tournament, so = . Therefore
2W +1 T
T (W + 2) = 2W +1 . Because T and W + 2 are both integers, it follows that both are powers of 2.
r
−1
9. From Problem 8, T (W + 2) = 2W +1 . Substitute in to obtain 2k · 2r = 22 . Taking logarithms base 2, it
follows that k + r = 2r − 1, so k = 2r − r − 1.
10. From the given information, it is possible that exactly one team will win its first t games, and will thus have
stake S(W, t, 0). Because t < W , this team has not yet won the tournament, so it must play at least one
more game. Let t0 be the number of wins its opponent in this game has at the time. If the opponent has no
losses, then t0 < k, because no other team wins its first t games. By the explanatory material, it follows that
S(W, t, 0) = S(W, t0 , 0). Compute directly that

W −t+1 W −t+2 W −t
S(W, t, 0) = − W −t+1 = W −t+1 .
2W −t 2 2
Now let x = W − t and x0 = W − t0 . Setting S(w, t, 0) = S(w, t0 , 0), substituting, and multiplying both sides
x x0
by 2 yields x = x0 . The function f (x) = 2xx is positive and strictly decreasing for x ≥ 2, so the only possible
2 2
solution to the above equation (involving x and x0 ), other than x = x0 , is (x, x0 ) = (1, 2), which gives t = W − 1
and t0 = W − 2. Because t0 < k, this is only possible if t = k = W − 1. Then S(W, t, 0) = S(W, W − 1, 0) = 41
from the formula above. This is also equal to S(W, W − 2, 1), so t0 = W − 2, as desired. Otherwise, the
opponent in the game discussed above must have 1 loss, and stake S(W, t0 , 1). Thus S(W, t, 0) = S(W, t0 , 1), as
desired.
W −t 1
11. Note that S(W, t, 0) = W −t+1
and S(W, t0 , 1) = W −t0 . From Problem 10, it follows that for each t with
2 2
0 W −t 1
k ≤ t < W , there exists some t with W −t+1 = W −t0 , and thus W − t must be a power of 2. Note that if
2 2
W − k ≥ 3, then W − k and W − (k + 1) are consecutive integers that are not 1 and 2, so they cannot both
be powers of 2, and therefore W − k ≤ 2. From Problem 9, it follows that W = 2r − 2 and k = 2r − r − 1, so
W − k = r − 1 ≤ 2, thus r ∈ {1, 2, 3}. If r = 1, then W = 0, which is not allowed. If r = 2, then W = 2 and
T = 2k = 2; this corresponds to the best-of-3 series discussed above. If r = 3, then W = 6 and T = 16. Thus
the only possible values of W are 2 and 6, as desired.
12. At the beginning of the tournament, play five “rounds” of games, in which each team plays one game against
another team with the same number of wins and losses.

• After one round, 8 teams have 1-0 records, and 8 teams are 0-1.
• After two rounds, 4 teams have 2-0 records, and 8 teams are 1-1. Thus 4 teams are eliminated.
• After three rounds, 2 teams have 3-0 records, and 6 teams are 2-1.
• After four rounds, one team is 4-0, and 4 teams are 3-1.
• In the fifth round, the 4-0 team does not play. Afterwards, one team (Team A) is 4-0, and two teams
(Teams B and C) are 4-1.

After that, have Team A play both B and C in succession.

ARML encourages the reproduction of our contest problems for non-commercial, educational purposes. 11
Commercial usage of ARML problems without permission and posting entire contests or contest books are prohibited.
• If Team A wins both games, then B and C are both eliminated, and A is 6-0 and wins the tournament.
• If Team A loses both games, then A is eliminated. Teams B and C are both 5-1; they play each other,
with the loser eliminated and the winner winning the tournament with a 6-1 record.
• If Team A wins one game, suppose without loss of generality that they beat B and lose to C. Then B is
eliminated, and A and C are both 5-1. They play each other, with the loser eliminated and the winner
winning the tournament with a 6-1 record.
Regardless of the results, the winner of the tournament has 6 wins, so the tournament is 6-perfect, as desired.

12 ARML encourages the reproduction of our contest problems for non-commercial, educational purposes.
Commercial usage of ARML problems without permission and posting entire contests or contest books are prohibited.
6 Individual Problems

Problem 1. Suppose that the set S has n elements. Given that there are eight times as many ways to choose a
three-element subset of S as there are ways to choose a two-element subset of S, compute n.

Problem 2. Circle Ω has radius 4, and AB is a diameter of Ω. Two circles of radius 1 are each internally tangent
to Ω and are tangent to AB at distinct points P and Q. Compute P Q.

Problem 3. Fabio created a sequence of integers where Fn = Fn−1 + Fn−2 for n > 2. He forgot what F1 and F2
were, but he remembered that F1 > 0 and that F11 = F1 . Compute the least possible value of F1 .

Problem 4. Let n be the number of ordered quadruples (A, B, C, D) such that each of A, B, √ C, and D is a subset
of {1, 2, 3, 4, 5, 6}, A ∩ B is a subset of C ∩ D, and A ∪ B is a subset of C ∪ D. Compute n.

Problem 5. Let a and b be positive real numbers, neither of which is equal to 1. Let x1 be the positive solution
√ √ √
to the equation ax = b x , and let x2 be the positive solution to the equation ( 3 a )x = (bc ) x , where c is a
x2
fixed positive real number. Given that = 2022, compute the integer closest to c.
x1

Problem 6. Five students—Ann, Ben, Cal, Dan, and Eve—are the only people attending a party. Each student
exchanges contact information with at most three other students. (Note that if P1 and P2 are two different
students attending the party and P1 has exchanged contact information with P2 , then this implies that P2 has
also exchanged contact information with P1 .) Compute the number of different possible sets of exchanges.

Problem 7. Compute the least positive integer n such that n! has two more digits than (n − 1)!.

Problem 8. In the diagram below, squares READ, T ALK, and W HIM have areas 81, 36, and 49, respectively,
with vertices M , I, and T on EA, AL, and AD, respectively. The entire figure is inscribed in square P U N S
as shown. Compute N P .

ARML encourages the reproduction of our contest problems for non-commercial, educational purposes. 13
Commercial usage of ARML problems without permission and posting entire contests or contest books are prohibited.
Problem 9. Compute the least positive odd integer that is not prime and does not divide 100!.

1 1
Problem 10. The decimal expansion of the fraction = consists of an infinitely repeating
6644349 27343 · 243
block of n digits. Compute the least possible value of n.

14 ARML encourages the reproduction of our contest problems for non-commercial, educational purposes.
Commercial usage of ARML problems without permission and posting entire contests or contest books are prohibited.
7 Answers to Individual Problems

Answer 1. 26

Answer 2. 4 2

Answer 3. 5

Answer 4. 1331

Answer 5. 15

Answer 6. 768

Answer 7. 15
114
Answer 8. (or 22 45 or 22.8)
5
Answer 9. 303

Answer 10. 2214

ARML encourages the reproduction of our contest problems for non-commercial, educational purposes. 15
Commercial usage of ARML problems without permission and posting entire contests or contest books are prohibited.
8 Solutions to Individual Problems

Problem 1. Suppose that the set S has n elements. Given that there are eight times as many ways to choose a
three-element subset of S as there are ways to choose a two-element subset of S, compute n.

Solution 1. The given conditions imply that


n
 n(n−1)(n−2)
3 3·2·1 n−2
n = = = 8,
2
n(n−1) 3
2·1

so n = 8 · 3 + 2 = 26.

Problem 2. Circle Ω has radius 4, and AB is a diameter of Ω. Two circles of radius 1 are each internally tangent
to Ω and are tangent to AB at distinct points P and Q. Compute P Q.

Solution 2. Let O be the center of Ω. Let ω denote the smaller circle tangent to Ω and whose point of tangency
is closest to A. Let P be the point where ω is tangent to AB.

Because ω is tangent to Ω, it follows that the distance between the centers of Ω and ω is 4 − 1 = 3. Also note
that a right triangle is formed by dropping a perpendicular from the center of ω to AB. This right √ triangle
has hypotenuse
√ 3 and a leg of length 1, so by the Pythagorean
√ √ Theorem,
√ the other leg has length 2 2. Thus
P O = 2 2, and by symmetry, P Q = P O + OQ = 2 2 + 2 2 = 4 2.

Problem 3. Fabio created a sequence of integers where Fn = Fn−1 + Fn−2 for n > 2. He forgot what F1 and F2
were, but he remembered that F1 > 0 and that F11 = F1 . Compute the least possible value of F1 .

Solution 3. Starting with the given recursion, F3 = F2 + F1 , F4 = F3 + F2 = 2F2 + F1 , F5 = F4 + F3 = 3F2 + 2F1 ,


and more generally, Fn can be expressed in terms of F2 and F1 , where the coefficients are consecutive Fibonacci
numbers. In particular, F11 = 55F2 +34F1 and also equals F1 , hence 55F2 = −33F1 , which implies 5F2 = −3F1 .
Each term in the sequence is an integer, so F1 is a positive multiple of 5, the least of which is 5.

16 ARML encourages the reproduction of our contest problems for non-commercial, educational purposes.
Commercial usage of ARML problems without permission and posting entire contests or contest books are prohibited.
Problem 4. Let n be the number of ordered quadruples (A, B, C, D) such that each of A, B, √ C, and D is a subset
of {1, 2, 3, 4, 5, 6}, A ∩ B is a subset of C ∩ D, and A ∪ B is a subset of C ∪ D. Compute n.

Solution 4. Consider the generalized problem where A, B, C, and D are subsets of S = {1, 2, . . . , m}. Given
the conditions on A, B, C, and D, each k ∈ S can be an element of any combination of those four subsets
except for: A only; B only; A and B only; A, B, and C only; and A, B, and D only. This leaves 11 choices
for the combination of subsets of which k can be an element. Each ordered quadruple (A, B, C, D) is uniquely
m
√ each k ∈ S, so it follows that the number of such quadruples is 11 . With m = 6,
determined by this choice for
it follows that n = 116 , so n = 113 = 1331.

Problem 5. Let a and b be positive real numbers, neither of which is equal to 1. Let x1 be the positive solution
√ √ √
to the equation ax = b x , and let x2 be the positive solution to the equation ( 3 a )x = (bc ) x , where c is a
x2
fixed positive real number. Given that = 2022, compute the integer closest to c.
x1

Solution 5. Substituting x1 into the first given equation and taking the base-a logarithm of each side yields
√ √
x1 = x1 · loga b. Because x1 is positive, divide each side by x1 and square each side to obtain x1 = (loga b)2 .
c 2
The second given equation is solved in the same manner, hence x2 = (log √3
a b ) . Thus

c ã2 ã2 ã2 ã2
log √ ab c log √ ab
Å Å Å Å
x2 3 3 loga b 1 1
= = = c2 √ · = c2 = (3c)2 .
x1 loga b loga b loga a loga b
3
1/3

The equation 9c2 = 2022 implies c2 = 224 23 ≈ 225, thus c ≈ 15, and the answer is 15.

Alternate Solution: Rewrite the two equations as



ax1 = b x1

ax2 /3 = b c x2
.

Solve for a by taking xth


1 and (x2 /3)
th
roots, respectively, to obtain
√1 √3c
a=b x1
=b x2
.

Because b 6= 1, setting the exponents of b equal implies


1 3c
√ =√ .
x1 x2

This means that


1 √

1 x2
c= · = · 2022.
3 x1 3
Because 2022 ≈ 2025 = 452 , it follows that c ≈ 45 ÷ 3 = 15.

Problem 6. Five students—Ann, Ben, Cal, Dan, and Eve—are the only people attending a party. Each student
exchanges contact information with at most three other students. (Note that if P1 and P2 are two different
students attending the party and P1 has exchanged contact information with P2 , then this implies that P2 has
also exchanged contact information with P1 .) Compute the number of different possible sets of exchanges.

Solution 6. There are 52 = 10 possible exchanges and thus 210 = 1024 possible sets of exchanges. Because there

are more sets of permissible exchanges than impermissible ones, use the inclusion-exclusion principle to count
the number of these sets that involve at least one student exchanging contact information with all four others.

ARML encourages the reproduction of our contest problems for non-commercial, educational purposes. 17
Commercial usage of ARML problems without permission and posting entire contests or contest books are prohibited.
If Ann exchanges information with all four others, then there are 6 other possible exchanges not involving
her, giving 26 = 64 possible combinations. There are 5 possible students who could be in Ann’s place, so in
total this encompasses 64 · 5 = 320 combinations of exchanges.

If Ann and Ben both exchange information  with all four others, then there are 3 other possible exchanges,
giving 23 = 8 combinations. There are 52 = 10 ways to choose which two students exchange information with
all four others, for a total of 8 · 10 = 80 combinations.

If Ann, Ben, and Cal all exchange information with everyone, then there is only one other possible exchange
(between Dan and Eve). This gives 2 · 53 = 20 combinations.


If Ann, Ben, Cal, and Dan all exchange information with everyone, then all ten exchanges must take place.
For the purpose of inclusion-exclusion, however, this case includes 54 = 5 combinations of exchanges. Finally,
if all five people exchange information with everyone, this gives one combination.

Using the inclusion-exclusion principle to account for the many combinations of exchanges that have been
counted multiple times in this process, one obtains 320 − 80 + 20 − 5 + 1 = 256 impermissible combinations,
and so the answer is 1024 − 256 = 768 different sets of possible exchanges.

Problem 7. Compute the least positive integer n such that n! has two more digits than (n − 1)!.

Solution 7. First note that 7! = 5040, and consider the following computations and approximations:

7! ≈ 5000 = 5 · 103
8! ≈ 40 · 103
9! ≈ 360 · 103
10! ≈ 3600 · 103
11! ≈ 39600 · 103 ≈ 4 · 107
12! ≈ 48 · 107 ≈ 5 · 108
13! ≈ 65 · 108
14! ≈ 910 · 108
15! ≈ 13650 · 108 .

Thus the answer is n = 15. Note that the exact values are 14! = 87178291200 and 15! = 1307674368000, which
confirms the estimation.

18 ARML encourages the reproduction of our contest problems for non-commercial, educational purposes.
Commercial usage of ARML problems without permission and posting entire contests or contest books are prohibited.
Problem 8. In the diagram below, squares READ, T ALK, and W HIM have areas 81, 36, and 49, respectively,
with vertices M , I, and T on EA, AL, and AD, respectively. The entire figure is inscribed in square P U N S
as shown. Compute N P .

Solution 8. Extend LK past K to meet N D at point X. Then drop a perpendicular from X to DT , meeting DT
at point Y .

Note that 4SDR, 4Y XD, 4N LX, 4IHL, and 4U W H are all similar. Let r = SD SR ; then the sides of each of

2
these triangles are in the ratio 1 : r : √1 + r . Because RD = 9 and XY = 6, and because XYAL is a rectangle,
2
it follows that SD = √1+r9r
2
, DX = 6 1+r r , DY = 6r , and AY = LX = 9 − 6r . This representation of the length
LX, together with the fact that HI = W H = 7, implies that N X = 9 − 6r · √1+r 1
, N L = 9 − 6r · √1+rr
 
2 2
,

7 1+r 2 √ 7
LH = r , and HU = 1+r 2
. Finally, recalling that U N = N S because P U N S is a square, it follows that
√ √
6 1 + r2 7 1 + r2
Å ã Å ã
9r 6 1 6 r 7
√ + + 9− ·√ = 9− ·√ + +√ .
1+r 2 r r 1+r 2 r 1+r 2 r 1 + r2

Multiply both sides by r 1 + r2 to obtain

9r2 + 6(1 + r2 ) + (9r − 6) = (9r2 − 6r) + 7(1 + r2 ) + 7r.

Solving the resulting quadratic equation yields r = 1 or r = 7. The former is extraneous because it would yield
AY = 3, which is less than AT . Therefore r = 7, which gives
√ √
63 6 50 57 1 57 2
SN = √ + + ·√ = .
50 7 7 50 5

Therefore N P = 2 · N S = 114 5 .

ARML encourages the reproduction of our contest problems for non-commercial, educational purposes. 19
Commercial usage of ARML problems without permission and posting entire contests or contest books are prohibited.
Problem 9. Compute the least positive odd integer that is not prime and does not divide 100!.

Solution 9. To find the least positive odd integer k satisfying the given conditions, consider values of k that are
the product of two odd primes, where one of these primes is greater than 100, and the other is a small odd
prime. The greater prime is at least 101, and the smaller prime is at least 3, so k ≥ 101 · 3 = 303, which indeed
satisfies the given conditions.

1 1
Problem 10. The decimal expansion of the fraction = consists of an infinitely repeating
6644349 27343 · 243
block of n digits. Compute the least possible value of n.

Solution 10. Note that if x1 has a repeating block of length n, then x1 = 10nk−1 for some integer k; by cross-
multiplying, it follows that x | 10n − 1. Thus the length of this repeating block is the least positive integer n
for which 6644349 | 10n − 1, or equivalently, 10n ≡ 1 (mod 6644349). (This number is called the order of 10
modulo 6644349, and in general, n satisfies this congruence if and only if it is divisible by the order.) Note
that 27343 = 303 + 73 = (30 + 7)(302 − 30 · 7 + 72 ) = 37 · 739, and 739 is prime. If e(x) denotes the order of 10
modulo x, then by the Chinese Remainder Theorem,

e(6644349) = lcm e(37), e(739), e(243) .

It remains to calculate each of the values in the above lcm expression.

First, note that e(37) = 3 because 103 − 1 = 999 = 9 · 111 = 33 · 37, and 37 does not divide either 102 − 1 = 99
or 101 − 1 = 9.

Second, by Fermat’s Little Theorem, 10738 ≡ 1 (mod 739), so the order of 10 modulo 739 must divide
738 = 2 · 32 · 41. One can verify that 10n 6≡ 1 (mod 739) for 1 ≤ n ≤ 9. Also, because 10n 6≡ −1 (mod 739) for
n = 6, 9, it follows that 10n 6≡ 1 (mod 739) for n = 12, 18, because the congruence x2 ≡ 1 (mod p) has only
the solutions x ≡ ±1 (mod p) for p prime. Thus e(739) is not a divisor of 18, and so it must be divisible by 41.

Now let a = 1041 . From the previous steps, note that 108 ≡ −2 (mod 739), from which a ≡ 10 · (−2)5 ≡
−320 ≡ 419 (mod 739), and a2 ≡ 100 · (−2)10 ≡ 102400 ≡ 418 (mod 739). In particular, a satisfies the con-
gruence a2 ≡ a − 1 (mod 739), from which a3 + 1 = (a + 1)(a2 − a + 1) ≡ 0 (mod 739). It follows that the
order of a modulo 739 is 6, and therefore e(739) = 41 · 6 = 246.

To find the order of 10 modulo 243, first note that if 10n ≡ 1 (mod 243), then 10n ≡ 1 (mod 81) as well,
n
and 81 divides 10n − 1 if and only if 9 divides 10 9−1 . This number consists (in base 10) of a string of n 1s,
whose digit sum is n. Thus the least n for which 81 divides 10n − 1 is n = 9, and in general, 81 divides 10n − 1
if and only if n is divisible by 9.
9
Now let n = 9m, and seek the least m for which 109m ≡ 1 (mod 243). Note that 1081−1 = 111111111
9 = 12345679,
which is congruent to 1 modulo 3. Thus 109 ≡ 82 (mod 243). Then, from the binomial expansion, it follows
that for some integer A,

109m ≡ (109 )m ≡ (81 + 1)m (mod 243)


2
≡ A · 81 + m · 81 + 1 (mod 243)
≡ 81m + 1 (mod 243).

(This technique is related to Hensel’s Lemma from number theory.) The least value of m for which this is
congruent to 1 modulo 243 is m = 3, so e(243) = 9 · 3 = 27.

Finally, e(6644349) = lcm(3, 246, 27) = 2 · 33 · 41 = 2214.

20 ARML encourages the reproduction of our contest problems for non-commercial, educational purposes.
Commercial usage of ARML problems without permission and posting entire contests or contest books are prohibited.

Alternate Solution: Proceed as above to deduce that e(6644349) = lcm e(37), e(739), e(243) and that
e(37) = 3.

To calculate e(243), note that in general, e(N ) must divide φ(N ) (where φ denotes Euler’s totient function),
and φ(243) = 2 · 34 . Trying the exponent 27, factor 1027 − 1 using the differences of cubes:

1027 − 1 = (109 − 1)(1018 + 109 + 1)


= (103 − 1)(106 + 103 + 1)(1018 + 109 + 1)
= (10 − 1)(102 + 10 + 1)(106 + 103 + 1)(1018 + 109 + 1)

which is divisible by 243. Therefore e(243) is a divisor of 27. Because 243 does not divide 109 − 1, in fact
e(243) = 27.

To calculate e(739), use the fact that e(739) divides 738 = 2 · 32 · 41. As before, manually check that no
divisor of 18 satisfies 10n ≡ 1 (mod 739), so e(739) is divisible by 41. It follows from the fact that e(243) = 27
that e(6644349) is divisible by 27, so the actual value of e(739) is not needed, only whether or not e(739) is even.

Let r be a primitive root modulo 739, meaning an integer whose order modulo 739 is 738. (A well-known
theorem of number theory states that a primitive root exists modulo any prime number.) If 10 ≡ rk (mod 739)
where k is as small as possible, then e(739) = 738 k . It follows that e(739) is odd if and only if k is even. In
turn, k is even if and only if 10 is congruent to a square modulo 739.

The Legendre symbol in number theory, ( ap ) (not a fraction), is defined as equaling 1 if a is congruent to
a square modulo a prime p, 0 if p | a, and −1 otherwise. A well-known theorem states that the Legendre
symbol is multiplicative, so Å ã Å ã Å ã
10 2 5
= · .
739 739 739
2

Another theorem states that 2 is congruent to a square modulo p if and only if p ≡ ±1 (mod 8), so 739 = −1.
Finally, a corollary of the Law of Quadratic Reciprocity gives
Å ã Å ã Å ã
5 739 4
= = = 1.
739 5 5
10

Therefore 739 = (−1)(1) = −1, which implies that e(739) is even.

Finally, it follows that e(6644349) = lcm(3, 2 · 41, 27) = 2 · 33 · 41 = 2214.

ARML encourages the reproduction of our contest problems for non-commercial, educational purposes. 21
Commercial usage of ARML problems without permission and posting entire contests or contest books are prohibited.
9 Relay Problems

Problem 1-1. Compute the least prime p such that (p − 1)! + 1 is divisible by two distinct primes.

Problem 1-2. Let T be the number you will receive. Jen has 5 blank cards and wishes to label them with 5 distinct
integers chosen from {1, 2, . . . , T }. However, she does not want the 5 labels to be 5 consecutive integers. For
example, {1, 2, 4, 5, 6} is valid (assuming T ≥ 6) while {2, 3, 4, 5, 6} is not. Compute the number of ways in
which Jen may choose the 5 labels under these conditions.

Problem 1-3. Let T be the number you will receive. Allan has a long row of boxes, each containing a card labelled
with some integer from {1, 2, . . . , T }. A wizard will trade Allan a special coin for either T cards labelled with
the same number or T cards with distinct number labels. Compute the least possible number of boxes that
Allan must open in order to guarantee that he can trade with the wizard twice to get two coins.

Problem 2-1. Compute the hundreds digit of 112022 .

3
Problem 2-2. Let T be the number you will receive. Compute the remainder when 17(T ) is divided by 100.

Problem 2-3. Let T be the number you will receive. Compute the remainder when 99T is divided by 1000.

22 ARML encourages the reproduction of our contest problems for non-commercial, educational purposes.
Commercial usage of ARML problems without permission and posting entire contests or contest books are prohibited.
10 Relay Answers

Answer 1-1. 7

Answer 1-2. 18

Answer 1-3. 308

Answer 2-1. 3

Answer 2-2. 73

Answer 2-3. 299

ARML encourages the reproduction of our contest problems for non-commercial, educational purposes. 23
Commercial usage of ARML problems without permission and posting entire contests or contest books are prohibited.
11 Relay Solutions

Problem 1-1. Compute the least prime p such that (p − 1)! + 1 is divisible by two distinct primes.

Solution 1-1. Note that (2−1)!+1 = 2, (3−1)!+1 = 3, and (5−1)!+1 = 25 = 5·5, while (7−1)!+1 = 721 = 7·103,
so 7 is the least such prime.

Problem 1-2. Let T be the number you will receive. Jen has 5 blank cards and wishes to label them with 5 distinct
integers chosen from {1, 2, . . . , T }. However, she does not want the 5 labels to be 5 consecutive integers. For
example, {1, 2, 4, 5, 6} is valid (assuming T ≥ 6) while {2, 3, 4, 5, 6} is not. Compute the number of ways in
which Jen may choose the 5 labels under these conditions.

Solution 1-2. There are T5 ways to choose the 5 numbers without discounting the consecutive integer sets. The


invalid sets are {1, 2, 3, 4, 5}, {2, 3, 4, 5, 6}, . . . , {T − 4, T − 3, T − 2, T − 1, T } for a total of T − 4 invalid sets.
Thus there are T5 − T + 4 valid sets. With T = 7, the answer is 18.


Problem 1-3. Let T be the number you will receive. Allan has a long row of boxes, each containing a card labelled
with some integer from {1, 2, . . . , T }. A wizard will trade Allan a special coin for either T cards labelled with
the same number or T cards with distinct number labels. Compute the least possible number of boxes that
Allan must open in order to guarantee that he can trade with the wizard twice to get two coins.

Solution 1-3. Assume that Allan trades in T cards for a coin whenever he is able to. Let N be the answer. By
definition, there must exist a sequence of cards that Allan gets such that after he has opened N − 1 boxes, he
has not traded twice yet. Note that he must have already traded once in this situation, otherwise he will have
to trade his N th card twice, which he cannot do. So Allan has N − 1 − T cards and one coin at this point.

These N − 1 − T cards must not be tradable. Thus there are at most T − 1 different labels among these cards,
and each of these labels can appear at most T −1 times. It follows that N −1−T ≤ (T −1)2 , so N ≤ T 2 −T +2.

But observe that it is possible to construct a sequence of cards that Allan gets so that he is only able to
trade after getting T 2 − T + 2 cards. Indeed, assume that Allan receives T − 1 cards of each label 1, 2, . . . , T − 2
in some order, followed by 2T cards of label T − 1. Then after (T − 1)2 + 1 cards, he trades in T cards of label
T − 1 for a coin, then after T more cards, he trades in T cards of label T − 1 again for another coin. With
T = 18, the answer is 308.

Problem 2-1. Compute the hundreds digit of 112022 .

Solution 2-1. Consider the binomial expansion of (a + b)2022 , where a = 10 and b = 1. Any term containing a3
or any higher power of a is irrelevant to the hundreds digit, leaving 2022·2021
2 a2 b2020 + 2022ab2021 + b2022 , or
1011 · 2021 · 100 + 20220 + 1. This number is congruent modulo 1000 to 100 + 220 + 1 = 321, so the hundreds
digit is 3.

3
Problem 2-2. Let T be the number you will receive. Compute the remainder when 17(T ) is divided by 100.

3
Solution 2-2. Consider the binomial expansion of (a + b)T , where a = 10 and b = 7. Any term containing a2 or
3 3
any higher power of a is congruent to 0 modulo 100, so ignore all such terms. This leaves T 3 abT −1 + bT =
3 3
10T 3 · 7T −1 + 7T . Now, note that 7x ≡ 7y mod 100 whenever x ≡ y mod 4. Because T 3 = 27, the desired
quantity is congruent modulo 100 to 10 · 27 · 72 + 73 , which is congruent to 10 · 7 · 9 + 43 ≡ 673 ≡ 73 (mod 100).

24 ARML encourages the reproduction of our contest problems for non-commercial, educational purposes.
Commercial usage of ARML problems without permission and posting entire contests or contest books are prohibited.
Problem 2-3. Let T be the number you will receive. Compute the remainder when 99T is divided by 1000.

Solution 2-3. Consider the binomial expansion of (a−b)T , where a = 100 and b = 1. Any term containing a2 or any
higher power of a is congruent to 0 modulo 1000, so ignore all such terms. This leaves T · 100 · (−1)T −1 + (−1)T .
With T = 73, this evaluates to 73 · 100 · (−1)72 + (−1)73 = 7300 − 1. Modulo 1000, this is 299.

ARML encourages the reproduction of our contest problems for non-commercial, educational purposes. 25
Commercial usage of ARML problems without permission and posting entire contests or contest books are prohibited.
12 Super Relay

4
1. Compute 20202 + 20200 + 25 .
2. Let T = TNYWR. The diagram at right shows two circles centered at M . Points
A and R lie on the larger circle and point L lies on the smaller circle so that chord
AR is tangent to M L at point L. Suppose that the area inside the larger circle and
outside the smaller circle (shaded in the diagram) is T π. Compute AR.

LE
3. Let T = TNYWR. Regular hexagon SILV ER is centered at point O, and IR = T . Compute .
OS
4. Let T = TNYWR, and let K = T 2 . Compute the sum of the tens and units digits of the product
 

(K − 2)(K − 1)(K)(K + 1)(K + 2).

5. Let T = TNYWR. A bag of jellybeans contains 20 purple jellybeans and T blue jellybeans. Celia randomly
chooses two jellybeans from the bag without replacement. Compute the probability that Celia’s two jellybeans
have different colors.
6. Let T = TNYWR. When T is expressed as a fraction in lowest terms, the numerator is K. Let a1 , a2 , a3 , . . .
be the arithmetic sequence with a1 = 25, a2 = 25 + K, a3 = 25 + 2K, etc. Compute the least positive integer n
such that an > 2022.
7. Let T = TNYWR, and let K be the sum of the distinct prime factors of T . The number log(2022 · K) can be
expressed in the form a + b log 2, where a and b are positive integers. Compute gcd(a, b).

15. Compute the value of K such that 12 is the minimum value of the function f (x) = Kx2 + 20x + 22.
14. Let T = TNYWR. Leon’s birthday is in the month of June (which has 30 days), but he decides to celebrate
it on a randomly chosen day D, in the month of June (i.e., on June Dth). Suppose that Leon’s birthday is
June 25. Compute the probability that |D − 25| < T . (That is, he celebrates his birthday less than T days
from his actual birthday.)
EF
13. Let T = TNYWR. In 4M AB, point E lies on M A and point F lies on M B so that EF k AB and = T.
  AB
AB [F CB]
Let point C lie on AB so that F C k M A. Compute − .
AC [M EF ]

q » p
12. Let T = TNYWR. Compute 20 + T 20 + T 20 + T 20 + · · · .

11. Let T = TNYWR. In pyramid DHAWS, base HAWS is a rectangle and point D lies in space so that
DH ⊥ HA, DH ⊥ HS, and D is not coplanar with base HAWS. The volume of pyramid DHAWS is 48,
HW = T , and DH = 3T − 3. Compute the length of the second longest edge of pyramid DHAWS.
10. Let T = TNYWR. Compute the number of positive divisors of lcm(T 6 , 603 ) that are perfect cubes.

9. Let T = TNYWR. Compute (1 − i)T /3 , where i = −1 .

8. Let A be the smallest digit in the number you will receive from position 7, and let B be the smallest digit in
the number you will receive from position 9. The graphs of y ≥ Ax − 20, y ≤ −B|x − 4| + 20, x ≥ 0, and y ≥ 0
define a bounded region in the coordinate plane. Compute the area of this region.

26 ARML encourages the reproduction of our contest problems for non-commercial, educational purposes.
Commercial usage of ARML problems without permission and posting entire contests or contest books are prohibited.
13 Super Relay Answers

1. 45

2. 6 5

3. 3
4. 2
40
5.
231
6. 51
7. 23

15. 10
1
14. (or 0.5)
2
13. 1
12. 5

11. 4 10
10. 24
9. 16

620
8. (or 206 23 or 206.6)
3

ARML encourages the reproduction of our contest problems for non-commercial, educational purposes. 27
Commercial usage of ARML problems without permission and posting entire contests or contest books are prohibited.
14 Super Relay Solutions

4
Problem 1. Compute 20202 + 20200 + 25 .

p √
Solution 1. Let T = 2020. Then the radicand√equals T 2 + 10T + 25 = (T + 5)2 . Thus 4
(T + 5)2 = T + 5.
Because T + 5 = 2025 = 452 , the answer is 452 = 45.

Problem 2. Let T = TNYWR. The diagram at right shows two circles centered at M . Points
A and R lie on the larger circle and point L lies on the smaller circle so that chord
AR is tangent to M L at point L. Suppose that the area inside the larger circle and
outside the smaller circle (shaded in the diagram) is T π. Compute AR.

Solution 2. Let M R = r1 and M L = r2 . Then the area of the shaded region is (r12 −r22 )π, so r12 −r22 = T . Because
p √
M L is perpendicular
√ to and bisects AR, it follows that AR = 2 · LR = 2 r12 − r22 = 2 T . With T = 45, the
answer is 6 5.

LE
Problem 3. Let T = TNYWR. Regular hexagon SILV ER is centered at point O, and IR = T . Compute .
OS

Solution 3. Let d be the side length of the hexagon. Note that drawing OS, OI, OL, OV , OE, and OR partitions
the hexagon into six equilateral triangles. Thus OS = d. The length LE can be calculated using 30◦ −60◦ −90◦
√ √ √ √
triangle geometry to be 2 · d 2 3 = d 3. Hence LE d 3
OS = d = 3 (independent of T ).

Let T = TNYWR, and let K = T 2 . Compute the sum of the tens and units digits of the product
 
Problem 4.
(K − 2)(K − 1)(K)(K + 1)(K + 2).

Solution 4. Because K is an integer and the product is of five consecutive integers, at least two of the factors
must√be even and exactly one factor must be a multiple of 5, so the units digit of the product is 0. With
T = 3, it follows that K = 3, and the product is 1 · 2 · 3 · 4 · 5 = 120, so the tens digit is 2, and the desired
sum is 2 + 0 = 2.

Problem 5. Let T = TNYWR. A bag of jellybeans contains 20 purple jellybeans and T blue jellybeans. Celia
randomly chooses two jellybeans from the bag without replacement. Compute the probability that Celia’s two
jellybeans have different colors.

20 T
Solution 5. Celia can either choose a purple jellybean and then a blue jellybean with probability 20+T · 19+T
T 20
or she can first choose a blue and then a purple jellybean with probability 20+T · 19+T . Thus the desired
probability is (20+T40T 80 40
)(19+T ) . With T = 2, this simplifies to 22·21 = 231 .

Problem 6. Let T = TNYWR. When T is expressed as a fraction in lowest terms, the numerator is K. Let
a1 , a2 , a3 , . . . be the arithmetic sequence with a1 = 25, a2 = 25 + K, a3 = 25 + 2K, etc. Compute the least
positive integer n such that an > 2022.

Solution 6. The given condition requires finding the least positive integer n such that 25 + (n − 1)K > 2022, so
n = 1997 40
 1997 2000
K + 1 . With T = 231 , K = 40, and the desired value of n is 40 + 1 = 40 + 1 = 51.

28 ARML encourages the reproduction of our contest problems for non-commercial, educational purposes.
Commercial usage of ARML problems without permission and posting entire contests or contest books are prohibited.
Problem 7. Let T = TNYWR, and let K be the sum of the distinct prime factors of T . The number log(2022 · K)
can be expressed in the form a + b log 2, where a and b are positive integers. Compute gcd(a, b).

Solution 7. Note that log(2022 · K) = log 1022 + log 222 + log K = 22 + 22 log 2 + log K. With T = 51, K =
3 + 17 = 20, so the desired expression can be written as 23 + 23 log 2. Thus a = b = 23, and gcd(a, b) = 23.

Problem 15. Compute the value of K such that 12 is the minimum value of the function f (x) = Kx2 + 20x + 22.

Solution 15. When a, b, and c are constants and a > 0, the graph of g(x) = ax2 + bx + c is a parabola pointing
upward. The minimum value of the function is the y-coordinate of the vertex, and the x-coordinate of the vertex
is −b −20 10 10 100 200 100

2a . Here, the x-coordinate of the vertex is 2K = − K and f − K = K − K + 22 = 12 → − K = −10,
so K = 10.

Problem 14. Let T = TNYWR. Leon’s birthday is in the month of June (which has 30 days), but he decides
to celebrate it on a randomly chosen day D, in the month of June (i.e., on June Dth). Suppose that Leon’s
birthday is June 25. Compute the probability that |D − 25| < T . (That is, he celebrates his birthday less than
T days from his actual birthday.)

Solution 14. If T is a positive integer, then in order for Leon to celebrate his birthday less than T days from his
actual birthday, he must celebrate it on any of the days max{1, 26 − T }, . . . , min{30, 24 + T }. With T = 10, he
must celebrate his birthday between June 16th and 30th, inclusive. This date range consists of 15 out of the
30 days in June. Thus the probability that |D − 25| < 10 is 15 1
30 = 2 .

Problem 13. Let T = TNYWR. In 4M AB, point E lies on M A and point   F lies on M B so that EF k AB and
EF AB [F CB]
= T . Let point C lie on AB so that F C k M A. Compute − .
AB AC [M EF ]

Solution 13. Because of the two given pairs of parallel segments, it follows that
4M EF ∼ 4M AB and quadrilateral AEF C is a parallelogram. Let
M B = x, M A = y, and AB = z. Then it follows that AC = EF = T z and
F B = (1 − T )x, F C = EA = (1 − T )y, and CB = AB − AC =q (1 − T )z.
AB z 1 [F CB] 1−T 2 AB [F CB]

Thus AC = T z = T and [M EF ] = T . Hence AC − [M EF ] =
1 1−T
T − T = 1 (independent of T ).


q » p
Problem 12. Let T = TNYWR. Compute 20 + T 20 + T 20 + T 20 + · · · .

Solution 12. Let the given expression equal x. Then x2 = 20 + T x. With T = 1, the equation x2 − x − 20 = 0
has solutions x = 5 or x = −4. Because x > 0, it follows that x = 5.

Problem 11. Let T = TNYWR. In pyramid DHAWS, base HAWS is a rectangle and point D lies in space so
that DH ⊥ HA, DH ⊥ HS, and D is not coplanar with base HAWS. The volume of pyramid DHAWS is 48,
HW = T , and DH = 3T − 3. Compute the length of the second longest edge of pyramid DHAWS.

Solution 11. Let SW = x and HS = y. Then 48 = 13 · DH · [SHAW ] = (T − 1)xy. Thus xy = T48 −1 and
2 2 2 2 2
x + y = T . With T = 5, it follows that DH = 12, x + y = 25, and xy = 12. By inspection, {x, y} = {3, 4}.
√ √
It quickly follows that the length of the second longest edge of the pyramid is 122 + 42 = 4 10.

ARML encourages the reproduction of our contest problems for non-commercial, educational purposes. 29
Commercial usage of ARML problems without permission and posting entire contests or contest books are prohibited.
Problem 10. Let T = TNYWR. Compute the number of positive divisors of lcm(T 6 , 603 ) that are perfect cubes.

Solution 10. Note that 60 = 22 · 31 · 51 . If p1 , p2 , . . . , pk are distinct primes, then the number of positive divisors of
√ √
pe11 pe22 · · · pkek that are perfect cubes is e31 + 1
    e2    ek  
3 + 1 ··· 3 + 1 . With T = 4 10 = 160, it follows
that T 6 = 1603 . Thus for any positive divisor of lcm(160, 60) = 25 · 31 · 51 , its cube will be a perfect cube divisor
of lcm(1603 , 603 ), and conversely. The number of positive divisors of 25 · 31 · 51 is (5 + 1)(1 + 1)(1 + 1) = 24,
so the answer is 24.


Problem 9. Let T = TNYWR. Compute (1 − i)T /3 , where i = −1 .

Solution 9. Note that (1 − i)2 = −2i, so (1 − i)4 = −4. Thus if T3 is an odd integer, then the answer will be
of the form (1 − i)m (−2i)n , where m = ±1 (depending on the sign of T ) and n is an integer. If T3 is an
T
even integer, then the answer will be of the form (−2i)n , where n is an integer. With T = 24, 3 = 8, so
(1 − i)8 = (−2i)4 = (−4)2 = 16.

Problem 8. Let A be the smallest digit in the number you will receive from position 7, and let B be the smallest
digit in the number you will receive from position 9. The graphs of y ≥ Ax − 20, y ≤ −B|x − 4| + 20, x ≥ 0,
and y ≥ 0 define a bounded region in the coordinate plane. Compute the area of this region.

Solution 8. Because A and B are digits, A and B are both nonnegative. Assuming B 6= 0, the graph of y ≤
−B|x − 4| + 20 looks like a downward pointing “V” shape along with all points that lie “inside” the V shape.
The vertex of this graph is (4, 20), the positive x-intercept is 4 + 20 B , 0 , and the y-intercept is (0, 20 − 4B).
Assuming A 6= 0, the graph of y ≥ Ax − 20 has x- and y-intercepts of 20

A , 0 and (0, −20), respectively. The
last intersection points to determine are where the graphs y = Ax − 20 and y = −B|x − 4| + 20 could intersect.
Note that the slope of the line containing (0, −20) (i.e., the y-intercept of the line y = Ax − 20) and (4, 20)
(i.e., the vertex of the absolute value graph) is 10. Thus if A > 10, then the line intersects Ä the absolute value ä
graph in the left branch, which is a ray of positive slope, and the intersection point is 40−4B A−B ,
20A+20B−4AB
A−B .
If A ≤ 10, then the line intersects Äthe absolute value graph ä in the right branch, which is a ray of negative
slope, and the intersection point is 40+4B A+B ,
20A−20B+4AB
A+B . With A = 2 (from 23) and B = 1 (from 16), the
line y = 2x − 20 intersects the x-axis and the graph of y = −|x − 4| + 20 in the points (10, 0) and 44 28

3 , 3 ,
respectively. Thus the region of interest is a pentagon with vertices (4, 20), (0, 16), (0, 0), (10, 0), and 44 28
3 , 3 ,
and the area can be determined using the Shoelace Formula:
4 · 16 + 0 · 0 + 0 · 0 + 10 · 28 + 44 · 20 − 20 · 0 + 16 · 0 + 0 · 10 + 0 · 44 + 28 · 4
 
3 3 3 3

1352 2
112

− 620
= 3 3
= .
2 3

30 ARML encourages the reproduction of our contest problems for non-commercial, educational purposes.
Commercial usage of ARML problems without permission and posting entire contests or contest books are prohibited.
15 Tiebreaker Problems

Problem 1. The sequence {an }n≥1 satisfies a1 = 1 and an = min{3n, 5n − an−1 } for n > 1. Compute the 2022nd
least positive integer K such that K 2 appears in the sequence.


Problem 2. Compute 20202 − 13442 − 13922 − 962 − 5282 − 2002 .

Problem 3. A four-digit positive integer is called consecucute if and only if the two-digit number formed by its
leftmost two digits is one less than the two-digit number formed by its rightmost two digits. Compute the sum
of all four-digit consecucute numbers.

ARML encourages the reproduction of our contest problems for non-commercial, educational purposes. 31
Commercial usage of ARML problems without permission and posting entire contests or contest books are prohibited.
16 Tiebreaker Answers

Answer 1. 2528

Answer 2. 20 21

Answer 3. 485495

32 ARML encourages the reproduction of our contest problems for non-commercial, educational purposes.
Commercial usage of ARML problems without permission and posting entire contests or contest books are prohibited.
17 Tiebreaker Solutions

Problem 1. The sequence {an }n≥1 satisfies a1 = 1 and an = min{3n, 5n − an−1 } for n > 1. Compute the 2022nd
least positive integer K such that K 2 appears in the sequence.

Solution 1. The first ten terms of the given sequence are 1, 6, 9, 11, 14, 16, 19, 21, 24, 26. One can show via
induction that this sequence comprises precisely the positive integers whose units digits are 1, 4, 6, 9, with
the exception of the value 4. Thus this sequence contains all positive squares except 22 and squares that are
multiples of 5. The 2022nd least integer K that is neither 2 nor a multiple of 5 is 2528.


Problem 2. Compute 20202 − 13442 − 13922 − 962 − 5282 − 2002 .

2 2 4 2 2 2
√ 2. Note that 2020 − 1344 = 3364 · 676 = 2 · 13 · 292 = 15082 , so the given expression
Solution is equal to
15082 − 13922 − 962 − 5282 − 200 2 . Similarly, note that 1508 − 1392 = 2900 · 116 = 24 · 52 · 292 = 5802 , so
√ 2 2
the given expression is equal to 5802 − 962 − 5282 − 2002 . Continuing √ in this way, note that 580 − 96 =
4 2 2 2 2 2 2
√ · 484 = 2 · 11
676 √ · 13 = 572 √ , so the given expression is equal to 572 − 528 − 200 . This is equal to
2202 − 2002 = 8400 = 20 21.

Problem 3. A four-digit positive integer is called consecucute if and only if the two-digit number formed by its
leftmost two digits is one less than the two-digit number formed by its rightmost two digits. Compute the sum
of all four-digit consecucute numbers.

Solution 3. The four-digit consecucute numbers are 1011, 1112, . . . , 9899, which is an 89-term arithmetic sequence
with common difference of 101. The sum of these terms is
89
(1011 + 9899) = 485495.
2

ARML encourages the reproduction of our contest problems for non-commercial, educational purposes. 33
Commercial usage of ARML problems without permission and posting entire contests or contest books are prohibited.

You might also like